Orthogonalité avec vecteurs (Ex..DM aidez moi)


  • L

    Voila j'ai mon DM a rendre pour mardi et jai encore un petit probleme.Jai repondu a pas mal de questionS mais un seule me pose un souci.. Je vous remercie d'avance si vous pouvez m'aider un petit peu.

    ABC est un triangle et O le centre de son cercle circonscrit. A' est le milieu du segment [BC], B' celui de [CA] et C' celui de [AB].

    On considere le point H défini par : →OH = →OA + →OB + →OC [1]

    1. Montrer que →OB + →OC = 2→OA'
    2. Deduire de la relation [1] que →AH= 2→OA'.
    3. Demontrer alors que les droites (AH) et (BC) sont perpendiculaires.
    4. De la meme maniere démontrer que la droite (BH) est perpendiculaire a la droite (AC).

    *Les question 1)2)3) je les ai réussies mais la 4) je bloque. *

    *Modification du titre par Zorro : parce qu'on demande (il suffit de lire les consignes) que les titres choisis soient explicites ! *


  • Zorro

    Bonjour,

    Quelle méthode as-tu utilisée pour montrer que les droites (AH) et (BC) sont perpendiculaires ?

    Ne peux-tu pas utiliser la même pour montrer que la droite (BH) est perpendiculaire a la droite (AC) ?


  • L

    Bonsoir,
    POur montrer que (AH) et (BC) sont perpendiculaire j'ai utiliser la loi qui dit " si deu droites sont paralleles alors toutes perpendiculaire a l'une l'est a l'autre". Mais pour (BH) et (AC) j'ai aucune parallele et aucune perpendiculaire... Je vois par contre que sur mon dessin (BH) est une hauteur pour le triangle ABC mais je n'ai rien qui me permet de le démontrer.


  • Zorro

    Je suis désolée, mais vue l'heure tardive, je ne vais pas commencer la figure maintenant. Je regarderai peut-être demain.


  • L

    Oui je comprends il n'y a aucun souci j'ai scanné la figureje pourrais l'envoyé demain. Bonne nuit !!


  • Thierry
    Modérateurs

    Salut,
    Si tu veux insérer la figure, nous te demandons, entre autres choses de donner les références du livre dont elle est extraite.


  • L

    Voici la figure: elle n'est pas extraite d'un livre c'est moi qui l'ai faite.

    http://img228.imageshack.us/img228/470/marion2sm7.jpg


  • Zorro

    Il suffit de lire l'énoncé :

    De la même manière démontrer que la droite (BH) est perpendiculaire à la droite (AC).

    Reprenons la méthode :

    -Déduire de la relation [1] que AH→^\rightarrow = 2OA'→^\rightarrow
    ici se pourrait être BH→^\rightarrow = 2O?'→^\rightarrow

    • Démontrer alors que les droites (AH) et (BC) sont perpendiculaires.
      ici se pourrait être démontrer alors que les droites (BH) et (AC) sont perpendiculaires.

  • L

    je ne vois vraiment pas comment exprimer le vecteur BH en fonction d'un vecteur O?


  • Zorro

    Mon ? dans mon dernier message n'était pas assez incitatif à la réflexion !!!!

    Déduire de la relation [1] que AH→^\rightarrow = 2OA'→^\rightarrow

    ici se pourrait être BH→^\rightarrow = 2OB'→^\rightarrow .... non ?


  • L

    Non je ne trouve rien. 😕


  • Zorro

    Mais c'est la même démarche que pour la première orthogonalité .... tout pareil .... tout identique .... il faut juste remplacer A par B et vive versa ....... dans les questions 2 et 3 et suivre le même raisonnement ....


  • L

    Merci beaucoup beaucoup ! J'aurais jamais trouvé ca de moi même ! merci encore ! Enfin j'avance un peu!


  • Zorro

    C'est un grand classique .... quand on te dit "Démontrer de la même manière que ..... c'est qu'il faut prendre le même raisonnement en regardant ce qui change entre la 1ère démonstration et la 2ème et faire les modifications suggérées par l'énoncé.


  • L

    Donc voici ma rédaction:

    Je sais que [OA] et [OC] sont des rayons du cercle donc ils sont de meme longueur. OA=OC
    OA→^\rightarrow +OC→^\rightarrow +AO→^\rightarrow +CO→^\rightarrow donne un quadrilatere avec ses cotés paralleles deux a deux et tous de meme longueur. Donc le quadrilatere est un losange. Or les diagonales d'un losange se coupent en leur milieu et perpendiculairement , donc B' est le point d'intersection des deux diagonales. Alors OA→^\rightarrow +OC→^\rightarrow = 2OB'→^\rightarrow

    OH→^\rightarrow = OA→^\rightarrow +OB→^\rightarrow +OC→^\rightarrow
    Comme OA→^\rightarrow +OC→^\rightarrow = 2 OB'→^\rightarrow alors on a :
    OH→^\rightarrow = 2OB'→^\rightarrow +OB→^\rightarrow
    AO→^\rightarrow +OH→^\rightarrow = 2OA'→^\rightarrow

    Je sais que (OB') est perpendiculaire a (AC) et que BH→^\rightarrow = 2OB'→^\rightarrow
    Donc BH→^\rightarrow est parallele a 2OA'→^\rightarrow . Or si deux droites sont paralleles alors toutes perpendiculaire a l'une l'est a l'autre donc (BH) est perpendiculaire a (AC)

    C'est correct si je rédige comme ca?


  • J

    Il faut juste que tu nommes le losange par les points qui le composent. Le losange est composé de O, A, C, et d'un autre que tu dois nommer.
    Voilà !


Se connecter pour répondre